ZFC vs ZF

JOSEPH SHIPMAN joeshipman at aol.com
Mon Sep 27 00:16:57 EDT 2021


Is there any well-known theorem of ZFC for which it is an open question whether it is a theorem of ZF?

Nothing involving large cardinals, please, I already know it is open whether you need choice to refute certain statements about embedding ranks into themselves.

— JS

Sent from my iPhone


More information about the FOM mailing list